Is the Parameterization Correct in Leithold's Stokes' Theorem Problem?

Click For Summary
The discussion centers on a problem from Leithold's calculus book regarding Stokes' theorem. A participant questions the parameterization of the curve, specifically whether it should be (x = 2 cos(t)) instead of (x = 5 cos(t)). The problem involves verifying Stokes' theorem for the function f(x,y) = y^2i + x^2j over a region bounded by the circle x^2 + y^2 = 4. The relevant equations for Stokes' theorem are provided, but the participant expresses confusion about their application. Clarification on the correct parameterization and its implications for the theorem's verification is sought.
runinfang
Messages
4
Reaction score
0
Homework Statement
Verify Stokes' theorem in the plane where f(x,y)=y2i+x2j
and the region formed is bounded by the circle (x^2+y^2=4).
Relevant Equations
Stokes' theorem in the plane
"Let \( M \) and \( N \) be the functions, \( C \) be the curve, and \( R \) be the region defined as in Green's theorem. If \( \mathbf{F}(x, y) = M(x, y)\mathbf{i} + N(x, y)\mathbf{j} \) and \( \mathbf{T}(s) \) is the unit tangent vector to \( C \) at \( P \), where \( s \) units is the arc length of \( C \) measured from a point \( P_0 \) to \( P \), then:

\[
\oint_C \mathbf{F} \cdot \mathbf{T} \, ds = \iint_R (\text{curl } \mathbf{F}) \cdot \mathbf{k} \, dA
\]"
The question is a problem from Leithold's calculus book. I didn't understand the (x = 5 \cos(t)). Shouldn't it be (x = 2 cos(t))? I'm referring to item b.
1710974059690.png

i tried this way. i don't know what is wrong.
1710975133627.png
 
Last edited:
Physics news on Phys.org
runinfang said:
Homework Statement: Verify Stokes' theorem in the plane where f(x,y)=y2i+x2j
and the region formed is bounded by the circle (x^2+y^2=4).
Relevant Equations: Stokes' theorem in the plane
"Let \( M \) and \( N \) be the functions, \( C \) be the curve, and \( R \) be the region defined as in Green's theorem. If \( \mathbf{F}(x, y) = M(x, y)\mathbf{i} + N(x, y)\mathbf{j} \) and \( \mathbf{T}(s) \) is the unit tangent vector to \( C \) at \( P \), where \( s \) units is the arc length of \( C \) measured from a point \( P_0 \) to \( P \), then:

\[
\oint_C \mathbf{F} \cdot \mathbf{T} \, ds = \iint_R (\text{curl } \mathbf{F}) \cdot \mathbf{k} \, dA
\]"

The question is a problem from Leithold's calculus book. I didn't understand the (x = 5 \cos(t)). Shouldn't it be (x = 2 cos(t))? I'm referring to item b.
View attachment 342076
i tried this way. i don't know what is wrong.
View attachment 342077
Please wrap your text with ## or otherwise to render Latex, make your text more readable.
 
Question: A clock's minute hand has length 4 and its hour hand has length 3. What is the distance between the tips at the moment when it is increasing most rapidly?(Putnam Exam Question) Answer: Making assumption that both the hands moves at constant angular velocities, the answer is ## \sqrt{7} .## But don't you think this assumption is somewhat doubtful and wrong?

Similar threads

  • · Replies 21 ·
Replies
21
Views
4K
  • · Replies 7 ·
Replies
7
Views
2K
  • · Replies 2 ·
Replies
2
Views
2K
  • · Replies 2 ·
Replies
2
Views
2K
  • · Replies 1 ·
Replies
1
Views
1K
  • · Replies 6 ·
Replies
6
Views
2K
Replies
7
Views
2K
  • · Replies 4 ·
Replies
4
Views
2K
  • · Replies 3 ·
Replies
3
Views
2K
  • · Replies 3 ·
Replies
3
Views
2K